JOIN ADRE 2.0 Telegram Group

Gynecology Quiz | Gynecology Multiple Choice Questions and Answers

Questions
1 A 35-year-old woman comes requesting long-term reversible contraception. You advise that the method that can provide the longest protection is:
A contraceptive implant
B copper intrauterine device
C Depo-Provera injection
D intrauterine hormonal system (IUS)

Answer: copper intrauterine device
2 A 25-year-old woman on liver enzyme inducers is requesting contraceptive advice. The method providing her with the most reliable form of contraception would be:
A combined oral contraceptive pill
B Depo-Provera injection
C diaphragm
D male condom

Answer: Depo-Provera injection
Advertisement
ADRE 2.0 Mock Test - 5 Start Test
ADRE 2.0 Mock Test - 5 Start Test

DOWNLOAD CURRENT AFFAIRS PDF FROM APP

3 One of the following is true. Decreased peripheral resistance in pregnancy has been attributed to an increase in synthesis of:
A angiotensin
B endothelin
C nitric oxide
D renin

Answer: nitric oxide
4 One of the following is true. The increase in maternal blood volume in pregnancy occurs as a result of:
A peripheral vasoconstriction
B a reduction in progesterone
C decreased synthesis of vasopressin
D increased aldosterone synthesis

Answer: increased aldosterone synthesis
5 One of the following is true. Successful fertilisation and subsequent normal embryonic development:
A require at least two spermatozoa
B require the retention of the cortical granules in the oocyte
C are most likely when the oocytes have been ovulated in an immature stage
D require exclusion of the second polar body

Answer: require exclusion of the second polar body
6 One of the following is true. It is recognised that the positive predictive value of initial mammography for breast cancer within the national screening programme in the UK is 16%. This means that:
A 16% of people who have breast cancer are detected on initial mammography
B 84% of people without breast cancer have a normal mammogram
C 16% of initial mammograms are abnormal
D a patient with an abnormal initial mammogram has a 16% chance of having breast cancer

Answer: a patient with an abnormal initial mammogram has a 16% chance of having breast cancer
7 Which is the most appropriate statement concerning pulmonary embolism?
A It is now rarely fatal, with the introduction of modern diagnostic tests and treatments.
B It gives an area of lung which is unventilated on a ventilationperfusion scan.
C It does not usually show up on a CT pulmonary angiogram.
D It may give symptoms similar to pneumonia.

Answer: It may give symptoms similar to pneumonia.
8 Which one of the following statements about puberty is true?
A Puberty is preceded by falling plasma levels of adrenal androgens.
B The fi rst menstrual period is called the adrenarche.
C The pubertal growth spurt is the fi rst sign of puberty.
D Pubic hair growth is stimulated in girls by oestrogen.

Answer: Puberty is preceded by falling plasma levels of adrenal androgens.
9 Which one of the following statements about the implantation of the human embryo is true?
A It will occur at any time over a period of about 14 days.
B It will occur whether or not the zona pellucida is present.
C It will occur when the cytotrophoblast contacts the endometrial epithelium and begins to invade the maternal tissue.
D It will occur with the inner cell mass closest to the endometrium.

Answer: It will occur with the inner cell mass closest to the endometrium.
10 Which of the following statements concerning the anterior pituitary is true?
A It develops in the embryo from a down-growth of the hypothalamus.
B It secretes antidiuretic hormone (ADH).
C It is regulated by hypothalamic-releasing hormones.
D It secretes its hormones into the pituitary portal system.

Answer: It is regulated by hypothalamic-releasing hormones.
11 Which one of the following statements about pituitary tumours is true?
A Weight loss is a common feature of pituitary failure (hypopituitarism) due to a pituitary tumour.
B Visual fi eld loss in female patients with prolactin-secreting pituitary tumours (prolactinoma) is usual.
C Adrenocorticotrophic hormone (ACTH) secreting pituitary tumours cause a syndrome of cortisol excess that can lead to exaggerated vertical growth in adolescence.
D A low testosterone level is more common than a low thyroxine level in men with non-functioning gonads.

Answer: A low testosterone level is more common than a low thyroxine level in men with non-functioning gonads.
12 The following are all causes of recurrent miscarriage, apart from:
A parental chromosomal abnormality
B activated protein C-resistance
C uncontrolled hypothyroidism
D chlamydia infection

Answer: chlamydia infection
13 The following are all consistent with the diagnosis of antiphospholipid syndrome except:
A hydatidiform mole
B severe early-onset pre-eclampsia
C arterial or venous thrombosis
D mid-trimester fetal loss

Answer: hydatidiform mole
14 A 20-year-old woman is referred with a problem of post-coital bleeding. Over the past two months it has occurred on six occasions and there has been a small amount of bright red blood noticed after intercourse. There is no associated pain. The following investigations should initially be performed, apart from:
A cervical smear
B endocervical swab for chlamydia
C colposcopy
D endocervical swab for gonorrhoea

Answer: colposcopy
15 The following statements regarding adenomyosis are true, apart from one.
A It tends to occur in women over 35 years.
B Risk factors include increased parity, termination and quick labours.
C The condition commonly occurs in association with endometriosis.
D With each period, bleeding occurs from the endometrial tissue into the smooth muscle.

Answer: Risk factors include increased parity, termination and quick labours.
16 A 24-year-old woman presents with the absence of periods for nine months. She started her periods at the age of 13 years and had a regular 28-day cycle until 18 months ago. The periods then became irregular, occurring every two to three months until they stopped completely. The following are all included in the differential diagnosis of secondary amenorrhoea, apart from:
A excessive exercise
B hyperprolactinaemia
C hyperthyroidism
D premature ovarian failure

Answer: hyperthyroidism
17 A 28-year-old woman attends the colposcopy clinic after an abnormal smear test. The smear is reported as severe dyskaryosis and she has an intrauterine contraceptive device in situ. All of the following statements are likely to be true, apart from:
A the cervix is macroscopically normal
B acetic acid is applied and an irregular white area is apparent to the left of the cervical os
C Lugol’s iodine is applied and the same area stains dark brown while the rest of the cervix stains pale
D a biopsy is taken

Answer: Lugol’s iodine is applied and the same area stains dark brown while the rest of the cervix stains pale
18 A 32-year-old woman presents to the gynaecology clinic with infrequent periods. A hormone profi le is done and all of the following are consistent with polycystic ovarian syndrome, apart from:
A increased androgen levels
B normal FSH
C normal oestradiol
D decreased LH

Answer: decreased LH
19 All of the following are effects of premature menopause, apart from:
A decreased cardiovascular risk
B infertility
C osteoporosis
D vasomotor symptoms

Answer: decreased cardiovascular risk
20 All of the following drugs are associated with hyperprolactinaemia, apart from:
A reserpine
B progesterone-only contraceptive pill
C methyldopa
D ranitidine

Answer: progesterone-only contraceptive pill

ADRE 2.0 MOCK TEST

Take Mock Tests

Government Schemes Mock Test Start Test!
Political Science Mock Test – 42 Start Test
History Test – 190 Start Test
Quantitative Aptitude Test Start Test!
Trigonometry - Mock Test Start Test!
Data Interpretation - Mock Test Start Test!
General Awareness - Mock Test Start Test!
Reasoning Ability - Mock Test Start Test!
Englist(Antonyms) Mock Test 1 Start Test!
Quantitative Aptitude (Percentage) Mock Test Start Test!
Economy Mock Test 1 Unlock Test!
Books & Authors - Test 2 Unlock Test!
Advertisement